A power series with decreasing positive coefficients has no zeroes in the disk












8












$begingroup$


Let $sum_{n=0}^infty a_n z^n$ be a formal complex power series, with $a_n$ strictly decreasing to 1 as $nto infty$. It is easy to see that the radius of convergence is 1, so that this power series represents a function $f$ holomorphic on the disk. The question is to



Show that f has no zeroes on the disk.



I've been looking at this a while and feel and tried a few different things.



$cdot$ One easy observation is the similarity of this series with $frac{1}{1-z}$, whose coefficients do not strictly decrease but which otherwise is an example of the thing to be proven.



$cdot$ Rouché's theorem doesn't seem to have an immediate application—I've tried comparing $f$ to $frac{1}{1-z}$ or to its partial sums by Rouché.



$cdot$ If the partial sums had no zeroes, then neither would $f$ by Hurwitz's theorem



$cdot$ If we could show the function had positive real part we'd obviously be done, which seems highly plausible—the condition gives that $f(-1) = sum a_n cospi n > 0$, but I don't know how to extend this argument to arguments other than $pi$. Besides, if this technique were to work, it seems it would rely most on algebraic manipulations of power series, and I would vastly prefer a classical complex analytic approach (argument principle, Rouché's theorem, etc)



Please advise!










share|cite|improve this question









$endgroup$

















    8












    $begingroup$


    Let $sum_{n=0}^infty a_n z^n$ be a formal complex power series, with $a_n$ strictly decreasing to 1 as $nto infty$. It is easy to see that the radius of convergence is 1, so that this power series represents a function $f$ holomorphic on the disk. The question is to



    Show that f has no zeroes on the disk.



    I've been looking at this a while and feel and tried a few different things.



    $cdot$ One easy observation is the similarity of this series with $frac{1}{1-z}$, whose coefficients do not strictly decrease but which otherwise is an example of the thing to be proven.



    $cdot$ Rouché's theorem doesn't seem to have an immediate application—I've tried comparing $f$ to $frac{1}{1-z}$ or to its partial sums by Rouché.



    $cdot$ If the partial sums had no zeroes, then neither would $f$ by Hurwitz's theorem



    $cdot$ If we could show the function had positive real part we'd obviously be done, which seems highly plausible—the condition gives that $f(-1) = sum a_n cospi n > 0$, but I don't know how to extend this argument to arguments other than $pi$. Besides, if this technique were to work, it seems it would rely most on algebraic manipulations of power series, and I would vastly prefer a classical complex analytic approach (argument principle, Rouché's theorem, etc)



    Please advise!










    share|cite|improve this question









    $endgroup$















      8












      8








      8


      2



      $begingroup$


      Let $sum_{n=0}^infty a_n z^n$ be a formal complex power series, with $a_n$ strictly decreasing to 1 as $nto infty$. It is easy to see that the radius of convergence is 1, so that this power series represents a function $f$ holomorphic on the disk. The question is to



      Show that f has no zeroes on the disk.



      I've been looking at this a while and feel and tried a few different things.



      $cdot$ One easy observation is the similarity of this series with $frac{1}{1-z}$, whose coefficients do not strictly decrease but which otherwise is an example of the thing to be proven.



      $cdot$ Rouché's theorem doesn't seem to have an immediate application—I've tried comparing $f$ to $frac{1}{1-z}$ or to its partial sums by Rouché.



      $cdot$ If the partial sums had no zeroes, then neither would $f$ by Hurwitz's theorem



      $cdot$ If we could show the function had positive real part we'd obviously be done, which seems highly plausible—the condition gives that $f(-1) = sum a_n cospi n > 0$, but I don't know how to extend this argument to arguments other than $pi$. Besides, if this technique were to work, it seems it would rely most on algebraic manipulations of power series, and I would vastly prefer a classical complex analytic approach (argument principle, Rouché's theorem, etc)



      Please advise!










      share|cite|improve this question









      $endgroup$




      Let $sum_{n=0}^infty a_n z^n$ be a formal complex power series, with $a_n$ strictly decreasing to 1 as $nto infty$. It is easy to see that the radius of convergence is 1, so that this power series represents a function $f$ holomorphic on the disk. The question is to



      Show that f has no zeroes on the disk.



      I've been looking at this a while and feel and tried a few different things.



      $cdot$ One easy observation is the similarity of this series with $frac{1}{1-z}$, whose coefficients do not strictly decrease but which otherwise is an example of the thing to be proven.



      $cdot$ Rouché's theorem doesn't seem to have an immediate application—I've tried comparing $f$ to $frac{1}{1-z}$ or to its partial sums by Rouché.



      $cdot$ If the partial sums had no zeroes, then neither would $f$ by Hurwitz's theorem



      $cdot$ If we could show the function had positive real part we'd obviously be done, which seems highly plausible—the condition gives that $f(-1) = sum a_n cospi n > 0$, but I don't know how to extend this argument to arguments other than $pi$. Besides, if this technique were to work, it seems it would rely most on algebraic manipulations of power series, and I would vastly prefer a classical complex analytic approach (argument principle, Rouché's theorem, etc)



      Please advise!







      complex-analysis power-series






      share|cite|improve this question













      share|cite|improve this question











      share|cite|improve this question




      share|cite|improve this question










      asked 14 hours ago









      Van LatimerVan Latimer

      363110




      363110






















          1 Answer
          1






          active

          oldest

          votes


















          9












          $begingroup$

          Let us consider $f(z) =(1-z)sumlimits_{n=0}^infty a_n z^n =a_0+sumlimits_{n=1}^infty (a_n-a_{n-1})z^n$. Suppose $f(re^{itheta})=0$ for $rle 1$. Then it follows
          $$
          a_0 =sum_{n=1}^infty (a_{n-1}-a_n)r^ne^{intheta},
          $$
          hence
          $$
          a_0=left|sum_{n=1}^infty (a_{n-1}-a_n)r^ne^{intheta}right|le sum_{n=1}^infty (a_{n-1}-a_n)r^nlesum_{n=1}^infty(a_{n-1}-a_n)=a_0-1,
          $$
          which leads to a contradiction. Since $f$ has no zeros on the unit disk, neither does $frac{f(z)}{1-z}=sumlimits_{n=0}^infty a_n z^n$.






          share|cite|improve this answer











          $endgroup$













          • $begingroup$
            That's beautiful, thank you. I looked at the series (1-z) sum a_n z^n but didn't see how to get this contradiction.
            $endgroup$
            – Van Latimer
            14 hours ago











          Your Answer





          StackExchange.ifUsing("editor", function () {
          return StackExchange.using("mathjaxEditing", function () {
          StackExchange.MarkdownEditor.creationCallbacks.add(function (editor, postfix) {
          StackExchange.mathjaxEditing.prepareWmdForMathJax(editor, postfix, [["$", "$"], ["\\(","\\)"]]);
          });
          });
          }, "mathjax-editing");

          StackExchange.ready(function() {
          var channelOptions = {
          tags: "".split(" "),
          id: "69"
          };
          initTagRenderer("".split(" "), "".split(" "), channelOptions);

          StackExchange.using("externalEditor", function() {
          // Have to fire editor after snippets, if snippets enabled
          if (StackExchange.settings.snippets.snippetsEnabled) {
          StackExchange.using("snippets", function() {
          createEditor();
          });
          }
          else {
          createEditor();
          }
          });

          function createEditor() {
          StackExchange.prepareEditor({
          heartbeatType: 'answer',
          autoActivateHeartbeat: false,
          convertImagesToLinks: true,
          noModals: true,
          showLowRepImageUploadWarning: true,
          reputationToPostImages: 10,
          bindNavPrevention: true,
          postfix: "",
          imageUploader: {
          brandingHtml: "Powered by u003ca class="icon-imgur-white" href="https://imgur.com/"u003eu003c/au003e",
          contentPolicyHtml: "User contributions licensed under u003ca href="https://creativecommons.org/licenses/by-sa/3.0/"u003ecc by-sa 3.0 with attribution requiredu003c/au003e u003ca href="https://stackoverflow.com/legal/content-policy"u003e(content policy)u003c/au003e",
          allowUrls: true
          },
          noCode: true, onDemand: true,
          discardSelector: ".discard-answer"
          ,immediatelyShowMarkdownHelp:true
          });


          }
          });














          draft saved

          draft discarded


















          StackExchange.ready(
          function () {
          StackExchange.openid.initPostLogin('.new-post-login', 'https%3a%2f%2fmath.stackexchange.com%2fquestions%2f3117764%2fa-power-series-with-decreasing-positive-coefficients-has-no-zeroes-in-the-disk%23new-answer', 'question_page');
          }
          );

          Post as a guest















          Required, but never shown

























          1 Answer
          1






          active

          oldest

          votes








          1 Answer
          1






          active

          oldest

          votes









          active

          oldest

          votes






          active

          oldest

          votes









          9












          $begingroup$

          Let us consider $f(z) =(1-z)sumlimits_{n=0}^infty a_n z^n =a_0+sumlimits_{n=1}^infty (a_n-a_{n-1})z^n$. Suppose $f(re^{itheta})=0$ for $rle 1$. Then it follows
          $$
          a_0 =sum_{n=1}^infty (a_{n-1}-a_n)r^ne^{intheta},
          $$
          hence
          $$
          a_0=left|sum_{n=1}^infty (a_{n-1}-a_n)r^ne^{intheta}right|le sum_{n=1}^infty (a_{n-1}-a_n)r^nlesum_{n=1}^infty(a_{n-1}-a_n)=a_0-1,
          $$
          which leads to a contradiction. Since $f$ has no zeros on the unit disk, neither does $frac{f(z)}{1-z}=sumlimits_{n=0}^infty a_n z^n$.






          share|cite|improve this answer











          $endgroup$













          • $begingroup$
            That's beautiful, thank you. I looked at the series (1-z) sum a_n z^n but didn't see how to get this contradiction.
            $endgroup$
            – Van Latimer
            14 hours ago
















          9












          $begingroup$

          Let us consider $f(z) =(1-z)sumlimits_{n=0}^infty a_n z^n =a_0+sumlimits_{n=1}^infty (a_n-a_{n-1})z^n$. Suppose $f(re^{itheta})=0$ for $rle 1$. Then it follows
          $$
          a_0 =sum_{n=1}^infty (a_{n-1}-a_n)r^ne^{intheta},
          $$
          hence
          $$
          a_0=left|sum_{n=1}^infty (a_{n-1}-a_n)r^ne^{intheta}right|le sum_{n=1}^infty (a_{n-1}-a_n)r^nlesum_{n=1}^infty(a_{n-1}-a_n)=a_0-1,
          $$
          which leads to a contradiction. Since $f$ has no zeros on the unit disk, neither does $frac{f(z)}{1-z}=sumlimits_{n=0}^infty a_n z^n$.






          share|cite|improve this answer











          $endgroup$













          • $begingroup$
            That's beautiful, thank you. I looked at the series (1-z) sum a_n z^n but didn't see how to get this contradiction.
            $endgroup$
            – Van Latimer
            14 hours ago














          9












          9








          9





          $begingroup$

          Let us consider $f(z) =(1-z)sumlimits_{n=0}^infty a_n z^n =a_0+sumlimits_{n=1}^infty (a_n-a_{n-1})z^n$. Suppose $f(re^{itheta})=0$ for $rle 1$. Then it follows
          $$
          a_0 =sum_{n=1}^infty (a_{n-1}-a_n)r^ne^{intheta},
          $$
          hence
          $$
          a_0=left|sum_{n=1}^infty (a_{n-1}-a_n)r^ne^{intheta}right|le sum_{n=1}^infty (a_{n-1}-a_n)r^nlesum_{n=1}^infty(a_{n-1}-a_n)=a_0-1,
          $$
          which leads to a contradiction. Since $f$ has no zeros on the unit disk, neither does $frac{f(z)}{1-z}=sumlimits_{n=0}^infty a_n z^n$.






          share|cite|improve this answer











          $endgroup$



          Let us consider $f(z) =(1-z)sumlimits_{n=0}^infty a_n z^n =a_0+sumlimits_{n=1}^infty (a_n-a_{n-1})z^n$. Suppose $f(re^{itheta})=0$ for $rle 1$. Then it follows
          $$
          a_0 =sum_{n=1}^infty (a_{n-1}-a_n)r^ne^{intheta},
          $$
          hence
          $$
          a_0=left|sum_{n=1}^infty (a_{n-1}-a_n)r^ne^{intheta}right|le sum_{n=1}^infty (a_{n-1}-a_n)r^nlesum_{n=1}^infty(a_{n-1}-a_n)=a_0-1,
          $$
          which leads to a contradiction. Since $f$ has no zeros on the unit disk, neither does $frac{f(z)}{1-z}=sumlimits_{n=0}^infty a_n z^n$.







          share|cite|improve this answer














          share|cite|improve this answer



          share|cite|improve this answer








          edited 4 hours ago

























          answered 14 hours ago









          SongSong

          14.1k1633




          14.1k1633












          • $begingroup$
            That's beautiful, thank you. I looked at the series (1-z) sum a_n z^n but didn't see how to get this contradiction.
            $endgroup$
            – Van Latimer
            14 hours ago


















          • $begingroup$
            That's beautiful, thank you. I looked at the series (1-z) sum a_n z^n but didn't see how to get this contradiction.
            $endgroup$
            – Van Latimer
            14 hours ago
















          $begingroup$
          That's beautiful, thank you. I looked at the series (1-z) sum a_n z^n but didn't see how to get this contradiction.
          $endgroup$
          – Van Latimer
          14 hours ago




          $begingroup$
          That's beautiful, thank you. I looked at the series (1-z) sum a_n z^n but didn't see how to get this contradiction.
          $endgroup$
          – Van Latimer
          14 hours ago


















          draft saved

          draft discarded




















































          Thanks for contributing an answer to Mathematics Stack Exchange!


          • Please be sure to answer the question. Provide details and share your research!

          But avoid



          • Asking for help, clarification, or responding to other answers.

          • Making statements based on opinion; back them up with references or personal experience.


          Use MathJax to format equations. MathJax reference.


          To learn more, see our tips on writing great answers.




          draft saved


          draft discarded














          StackExchange.ready(
          function () {
          StackExchange.openid.initPostLogin('.new-post-login', 'https%3a%2f%2fmath.stackexchange.com%2fquestions%2f3117764%2fa-power-series-with-decreasing-positive-coefficients-has-no-zeroes-in-the-disk%23new-answer', 'question_page');
          }
          );

          Post as a guest















          Required, but never shown





















































          Required, but never shown














          Required, but never shown












          Required, but never shown







          Required, but never shown

































          Required, but never shown














          Required, but never shown












          Required, but never shown







          Required, but never shown







          Popular posts from this blog

          Hivernacle

          Fluorita

          Hulsita